Đến nội dung

Khoa Linh nội dung

Có 599 mục bởi Khoa Linh (Tìm giới hạn từ 25-04-2020)



Sắp theo                Sắp xếp  

#709210 $\boxed{\text{Chuyên Đề}}$ Bất đẳng thức - Cực trị

Đã gửi bởi Khoa Linh on 24-05-2018 - 20:33 trong Bất đẳng thức và cực trị

 

Cho 3 số a,b,c thỏa mãn:0 ≤ c ≤ b ≤ a ≤ 2 và a+b+c=3.

Tìm GTLN của a3 + b3 + c3

 

Ta có: 

$3a\geq a+b+c=3\Rightarrow 1\leq a\leq 2\Leftrightarrow (a-1)(a-2)\leq 0$

Ta lại có: 

$a^3+b^3+c^3=a^3+(b+c)^3-3bc(b+c)\leq a^3+(3-a)^3=9(a^2-3a+2)+9=9(a-1)(a-2)+9\leq 9$




#701385 $\boxed{\text{Chuyên Đề}}$ Bất đẳng thức - Cực trị

Đã gửi bởi Khoa Linh on 08-02-2018 - 21:58 trong Bất đẳng thức và cực trị

Cho x,y>0 , x+y = 3 . Tìm GTNN của P = $2x^{2}+y^{2}+\frac{28}{x}+\frac{1}{y}$

Bài này chọn điểm rơi x=2, y=1 là ra thôi 

$\left (2x^2+8 \right )+(y^2+1)+\frac{28}{x}+\frac{1}{y}-9 \geq 8x+2y+\frac{28}{x}+\frac{1}{y}-9=(7x+\frac{28}{x})+(y+\frac{1}{y})+(x+y)-9\geq 28+2+3-9=24$




#700246 Phương Pháp : Đặt ẩn phụ đưa về hệ phương trình đối xứng loại $2$

Đã gửi bởi Khoa Linh on 13-01-2018 - 20:29 trong Phương trình, hệ phương trình và bất phương trình

M.n cho mình hỏi với phương trình này thì đặt kiểu gì :

$x^{2}-8x+9=\sqrt{16x-24}$

 

$\large PT\Leftrightarrow (x-3)^2-2x=2\sqrt{2(x-3)+2x}$

Đặt $\large \sqrt{2(x-3)+2x}=a;x-3=b\Rightarrow 2b+2x=a^2;2a+2x=b^2$




#707293 [TOPIC] ÔN THI BẤT ĐẲNG THỨC $\boxed{\text{THPT CHUYÊN}}$...

Đã gửi bởi Khoa Linh on 29-04-2018 - 15:25 trong Tài liệu - Đề thi

Bài 107:

Cho 3 số thực $a, b, c$ thỏa mãn $a+b+c=1$. Chứng minh:

$\frac{a}{{{a^2} + 1}} + \frac{b}{{{b^2} + 1}} + \frac{c}{{{c^2} + 1}} \le \frac{9}{{10}}$(Poland MO 1992)

Theo Dirichlet ta giả sử $(3b-1)(3c-1)\geq 0\Leftrightarrow b^2+c^2\leq \frac{1}{9}+\left ( b+c-\frac{1}{3} \right )^2=\frac{1}{9}+\left ( \frac{2}{3}-a \right )^2$

Ta có BĐT tương đương với: 

$\frac{a}{a^2+1}\leq \left ( \frac{1}{2}-\frac{b}{b^2+1} \right )+\left (\frac{1}{2}-\frac{c}{c^2+1} \right )-\frac{1}{10}\Leftrightarrow \frac{(b-1)^2}{b^2+1}+\frac{(c-1)^2}{c^2+1}\geq \frac{1}{5}+\frac{2a}{a^2+1}$

Mặt khác theo Cauchy - Schwarz ta có:

$\frac{(b-1)^2}{b^2+1}+\frac{(c-1)^2}{c^2+1}\geq\frac{(b+c-2)^2}{b^2+c^2+2}=\frac{(1+a)^2}{(b^2+c^2)+2}\geq \frac{(1+a)^2}{\frac{1}{9}+2+\left ( \frac{2}{3}-a \right )^2}=\frac{9(a+1)^2}{23-12a+9a^2}$

Vây ta đưa BĐT về BĐT 1 biến: 

$\frac{9(1+a)^2}{23-12a+9a^2}\geq \frac{a^2+10a+1}{5(1+a^2)}\Leftrightarrow (3a-1)^2(2a^2+2a+11)\geq 0$

Vậy ta có đpcm

p/s: Anh tr2512 xem lại hộ em nguồn bài toán bởi vì em làm 1 lần rồi và nó ghi là 1996 chứ không phải 1992. Em cảm ơn ...




#707149 [TOPIC] ÔN THI BẤT ĐẲNG THỨC $\boxed{\text{THPT CHUYÊN}}$...

Đã gửi bởi Khoa Linh on 27-04-2018 - 23:53 trong Tài liệu - Đề thi

Bài 105: Cho $a,b,c$ thỏa mãn $a\geq 4;b\geq 5,7\geq c\geq 6$ và $a^{2}+b^{2}+c^{2}=90$. Tìm giá trị nhỏ nhất $a+b+c$

 

P/s: Có 2 bài 103, nhờ DOTOANNANG sửa lại số thứ tự bài

Bài này chỉ cần điều kiện $c\geq 6$ thôi bởi vì từ giả thiết 

Ta có: 

$a\geq 4;b\geq 5\Rightarrow c\leq 7\Rightarrow (c-6)(c-7)\leq 0\Leftrightarrow 13c\geq c^2+42$

$b\geq 5;c\geq 6\Rightarrow a<9\Rightarrow (a-4)(a-9)\leq 0\Leftrightarrow 13a\geq a^2+36$

$a\geq 4;c\geq 6\Rightarrow b<8\Rightarrow (b-5)(b-8)\leq 0\Leftrightarrow 13b\geq b^2+40$

Suy ra $13(a+b+c)\geq (a^2+b^2+c^2)+42+36+40\Leftrightarrow a+b+c\geq 16$




#707600 [TOPIC] ÔN THI BẤT ĐẲNG THỨC $\boxed{\text{THPT CHUYÊN}}$...

Đã gửi bởi Khoa Linh on 03-05-2018 - 22:38 trong Tài liệu - Đề thi

Bài 119: Cho các số dương a, b, c sao cho $a+b+c+2=abc$. Chứng minh rằng:

$\frac{a}{b+1}+\frac{b}{c+1}+\frac{c}{a+1}\geq 2$

(Sưu tầm)




#707228 [TOPIC] ÔN THI BẤT ĐẲNG THỨC $\boxed{\text{THPT CHUYÊN}}$...

Đã gửi bởi Khoa Linh on 28-04-2018 - 22:50 trong Tài liệu - Đề thi

105. Cho $a\,,b\,\,> 0$. Chứng minh rằng:

 

$\frac{1}{2 + a + b} + \frac{a}{2a + b + 1} + \frac{b}{2b + a + 1} \,\, \leqq \frac{3}{4}$

Anh viet9a14124869 lời giải bài này đâu cần phức tạp quá vậy ạ... 

Ta có:

$\frac{4}{a+b+2}+\frac{4a}{2a+b+1}+\frac{4b}{2b+a+1}=\frac{4}{(a+1)+(b+1)}+\frac{4a}{(a+b)+(a+1)}+\frac{4b}{(a+b)+b+1}\leq \frac{1}{a+1}+\frac{1}{b+1}+\frac{a}{a+b}+\frac{a}{a+1}+\frac{b}{a+b}+\frac{b}{b+1}=3$




#708045 [TOPIC] ÔN THI BẤT ĐẲNG THỨC $\boxed{\text{THPT CHUYÊN}}$...

Đã gửi bởi Khoa Linh on 10-05-2018 - 20:36 trong Tài liệu - Đề thi

Bài 124: Cho a,b,c>0. Chứng minh rằng :

                                 $\frac{a^{3}}{(a+b)^{3}}+ \frac{b^{3}}{(b+c)^{3}}+\frac{c^{3}}{(a+c)^{3}}\geq \frac{3}{8}$

                                                                                     (Đề thi chọn đội tuyển quốc gia dự thi IMO 2005)

Ta viết lại BĐT dưới dạng 

$\sum \frac{1}{\left ( 1+\frac{b}{a} \right )^3}\geq \frac{3}{8}$

Đặt $\frac{b}{a}=x;\frac{c}{b}=y;\frac{a}{c}=z\Rightarrow xyz=1$

Áp dụng AM- GM ta có:

$\frac{1}{(1+x)^3}+\frac{1}{(1+x)^3}+\frac{1}{8}\geq \frac{3}{2}.\frac{1}{(x+1)^2}$

Thiết lập các BĐT tương tự ta quy về chứng minh:

$\frac{1}{(x+1)^2}+\frac{1}{(y+1)^2}+\frac{1}{(z+1)^2}\geq \frac{3}{4}$

Áp dụng BĐT phụ sau ta có:

$\frac{1}{(1+x)^2}+\frac{1}{(1+y)^2}+\frac{1}{(1+z)^2}\geq \frac{1}{xy+1}+\frac{1}{(1+z)^2}\geq \frac{3}{4}\Leftrightarrow \frac{z}{z+1}+\frac{1}{(1+z)^2}\geq \frac{3}{4}\Leftrightarrow \left ( \frac{1}{z+1}-\frac{1}{2} \right )^2\geq 0$

Suy ra ĐPCM




#710047 [TOPIC] ÔN THI BẤT ĐẲNG THỨC $\boxed{\text{THPT CHUYÊN}}$...

Đã gửi bởi Khoa Linh on 05-06-2018 - 19:40 trong Tài liệu - Đề thi

Bên đó thi chuyên chưa?

$\boxed{\text{Bài 143}}$ Cho $a,b,c >0$ ;abc=1.

Chứng minh $\frac{1}{(1+a)^2}+\frac{1}{(1+b)^2}+\frac{1}{(1+c)^2}+\frac{2}{(1+a)(1+b)(1+c)} \geq 1$

 

 

Lời giải của mình khá "vất vả" nhưng đem lại hiệu quả: 

Quy đồng ta có BĐT tương đương:

$\sum (a+1)^2(b+1)^2+2(a+1)(b+1)(c+1)\geq (a+1)^2(b+1)^2(c+1)^2$

$\Leftrightarrow (a+b+c)^2+4(a+b+c)-6abc-a^2b^2c^2-2abc(ab+bc+ca)-4abc(a+b+c)+4 \geq 0$

Thay $abc=1$ ta có BĐT trở thành: $a^2+b^2+c^2\geq 3$ (đúng theo AM-GM)

p/s: Bên mình sắp thi rồi, mùng 7,8 tháng 6. Mong bạn sớm đưa ra lời giải hay hơn  :D  :D




#710011 [TOPIC] ÔN THI BẤT ĐẲNG THỨC $\boxed{\text{THPT CHUYÊN}}$...

Đã gửi bởi Khoa Linh on 05-06-2018 - 14:45 trong Tài liệu - Đề thi

Bài 142: Cho 3 số thực a, b, c thỏa mãn điều kiện: a + b + c + ab + bc + ca = 6

Chứng minh rằng a2 + b2 + c2 ≥ 3

Ta có:

$12=2a+2b+2c+2ab+2bc+2ca\leq (a^2+1)+(b^2+1)+(c^2+1)+(a^2+b^2)+(b^2+c^2)+(c^2+a^2)\Leftrightarrow 12\leq 3+3(a^2+b^2+c^2)\Leftrightarrow 3\leq a^2+b^2+c^2$

p/s: Bài này quá cơ bản rồi 




#709473 [TOPIC] ÔN THI BẤT ĐẲNG THỨC $\boxed{\text{THPT CHUYÊN}}$...

Đã gửi bởi Khoa Linh on 29-05-2018 - 00:26 trong Tài liệu - Đề thi

Cháy lên nào TOPIC ơi

$\boxed{\text{Bài 135}}$ Cho $a,b,c$ là các số thực dương. Chứng minh rằng

$\frac{8}{a+b)^2+4abc}+\frac{8}{(b+c)^2+4abc}+\frac{8}{(c+a)^2+4abc} +a^2+b^2+c^2 \geq \frac{8}{a+3}+\frac{8}{b+3}+\frac{8}{c+3}$

 

Ta có:

$\frac{8}{(a+b)^2+4abc}+\frac{a^2+b^2}{2}\geq \frac{8}{(a+b)^2(c+1)}+\frac{(a+b)^2}{4}\geq \frac{4}{\sqrt{2(c+1)}}\geq \frac{8}{c+3}$

Thiết lập các BĐT còn lại ta có đpcm




#707024 [TOPIC] ÔN THI BẤT ĐẲNG THỨC $\boxed{\text{THPT CHUYÊN}}$...

Đã gửi bởi Khoa Linh on 26-04-2018 - 18:34 trong Tài liệu - Đề thi

Bài 101:

Cho a, b, c là các số thực dương. Chứng minh:

$$ \frac{a^2}{b} + \frac{b^2}{c} + \frac{c^2}{a} \ge \sqrt{\frac{a^2+b^2}{2}}+\sqrt{\frac{b^2+c^2}{2}}+\sqrt{\frac{a^2+c^2}{2}} \text{  (Sưu tầm)}$$

P/s: Bài này bị gọi là quá sức không :D theo mình là không

Bài này có đăng trong TTT2, lời giải không quá phức tạp nhưng do vẫn còn hạn gửi bài cho tòa soạn nên đề nghị mọi người không giải...




#707704 [TOPIC] ÔN THI BẤT ĐẲNG THỨC $\boxed{\text{THPT CHUYÊN}}$...

Đã gửi bởi Khoa Linh on 05-05-2018 - 18:14 trong Tài liệu - Đề thi

Bài 121: Cho các số thực dương x,y,z thay đổi thỏa mãn : $x(x+1)+y(y+1)+z(z+1)\leq 18$

Tìm GTNN của biểu thức: A=$\frac{1}{x+y+1}+\frac{1}{y+z+1}+\frac{1}{z+x+1}$

Ta có:

$18\geq x(x+1)+y(y+1)+z(z+1)=x^2+y^2+z^2+x+y+z\Leftrightarrow 30\geq (x^2+4)+(y^2+4)+(z^2+4)+(x+y+z)\geq 5(x+y+z)\Rightarrow x+y+z\leq 6$

Suy ra:

$A=\sum \frac{1}{x+y+1}\geq \frac{9}{2(x+y+z)+3}\geq \frac{3}{5}$




#706752 [TOPIC] ÔN THI BẤT ĐẲNG THỨC $\boxed{\text{THPT CHUYÊN}}$...

Đã gửi bởi Khoa Linh on 23-04-2018 - 17:29 trong Tài liệu - Đề thi

Bài 79: Cho a, b, c là các số thực dương thỏa mãn $a+b+c=3$. CMR:

$(a^2-a+1)(b^2-b+1)(c^2-c+1)\geq 1$

Cách 1(Dirichlet):

Giả sử $(b-1)(c-1)\geq 0\Rightarrow (b^2-b+1)(c^2-c+1)=bc(b-1)(c-1)+b^2+c^2-b-c+1\geq \frac{(b+c)^2}{2}-(b+c)+1=\frac{(3-a)^2}{2}-(3-a)+1=\frac{a^2-4a+5}{2}$

Vậy ta cần đi chứng minh: 

$(a^2-a+1)(a^2-4a+5)\geq 2\Leftrightarrow (a-1)^2)(a^2-3a+3)\geq 0$ luôn đúng

Cách 2(Cauchy - Schwarz):

Giả sử $c=min\left \{ a,b,c \right \}\Rightarrow c\leq 1$

Áp dụng Cauchy - Schwarz ta có:

$(a^2-a+1)(b^2-b+1)=\left ( (a-\frac{1}{2})^2+\frac{1}{2}+\frac{1}{4} \right )\left ( \frac{1}{4}+\frac{1}{2}+(b-\frac{1}{2})^2 \right )\geq\left [ \frac{1}{2}\left ( a-\frac{1}{2} \right )+\frac{1}{2}\left (b-\frac{1}{2} \right )+\frac{1}{2} \right ]^2=\frac{(3-c)^2}{4}$

Ta cần đi chứng minh: 

$(c^2-c+1)(3-c)^2\geq 4\Leftrightarrow (c-1)^2(c^2-5c+5)\geq 0$ luôn đúng do giả sử $c\leq 1$




#706315 [TOPIC] ÔN THI BẤT ĐẲNG THỨC $\boxed{\text{THPT CHUYÊN}}$...

Đã gửi bởi Khoa Linh on 18-04-2018 - 19:36 trong Tài liệu - Đề thi

Bài 35: Cho $\left\{\begin{matrix} x,y,z>0 & & \\ \frac{1}{x}+\frac{1}{y}+\frac{1}{z}=4 \end{matrix}\right.$ Tìm GTLN của biểu thức:

                       P= $\frac{1}{\alpha x+\beta y+\gamma z}+\frac{1}{\beta x+\gamma y+\alpha z}+\frac{1}{\gamma x+\alpha y+\beta z}$ với $\alpha ,\beta ,\gamma \in N^{*}$.

alpha, beta, gamma lằng nhằng quá :)) mình thay bằng a,b,c. Ta có:

$\frac{1}{ ax+b y+c z}=\frac{\frac{1}{a}+\frac{1}{b}+\frac{1}{c}}{(ax+by+cz)\left ( \frac{1}{a}+\frac{1}{b}+\frac{1}{c} \right )}\leq \left ( \frac{1}{a}+\frac{1}{b}+\frac{1}{c} \right )\left ( \frac{1}{\sqrt{x}+\sqrt{y}+\sqrt{z}} \right )^2$

Tương tự thì 

$P\leq 3\left ( \frac{1}{a}+\frac{1}{b}+\frac{1}{c} \right )\left ( \frac{1}{\sqrt{x}+\sqrt{y}+\sqrt{z}} \right )^2\leq \frac{3}{81}\left( \frac{1}{a}+\frac{1}{b}+\frac{1}{c} \right )\left ( \frac{1}{\sqrt{x}}+\frac{1}{\sqrt{y}}+\frac{1}{\sqrt{z}} \right )^2$

Mặt khác:

$\sum \frac{1}{\sqrt{x}}=\sum \frac{\sqrt{\frac{1}{x}.\frac{4}{3}}}{\sqrt{\frac{4}{3}}}\leq \frac{\sum\frac{1}{x}+4 }{2\sqrt{\frac{4}{3}}}=2\sqrt{3}$

Suy ra P$\leq\frac{4}{9}\left (\frac{1}{a}+\frac{1}{b}+\frac{1}{c} \right )$

p/s: Bạn xem lại điểm rơi nhé sợ mình làm sai nhưng ý tưởng là vậy 




#706242 [TOPIC] ÔN THI BẤT ĐẲNG THỨC $\boxed{\text{THPT CHUYÊN}}$...

Đã gửi bởi Khoa Linh on 17-04-2018 - 23:00 trong Tài liệu - Đề thi

Bài 28(IMO 1961): Cho tam giác ABC có độ dài 3 cạnh là a, b, c và có diện tích là S. Chứng minh rằng: $a^2+b^2+c^2\geq 4\sqrt{3}S$

Bất đẳng thức tương đương:

$\left ( a^2+b^2+c^2 \right )^2\geq 48p(p-a)(p-b)(p-c)\Leftrightarrow (a^2+b^2+c^2)^2\geq 3(a+b+c)(a+b-c)(b+c-a)(c+a-b)$

Theo AM - GM và Schur ta có:

$(a^2+b^2+c^2)^2\geq \frac{(a+b+c)^4}{9}=3(a+b+c).\frac{(a+b+c)^3}{27}\geq 3(a+b+c).abc\geq 3(a+b+c)(a+b-c)(b+c-a)(c+a-b)$




#706237 [TOPIC] ÔN THI BẤT ĐẲNG THỨC $\boxed{\text{THPT CHUYÊN}}$...

Đã gửi bởi Khoa Linh on 17-04-2018 - 22:43 trong Tài liệu - Đề thi

Bài 27: Cho a, b, c là các số không âm thỏa mãn: $a+b+c=3$

Chứng minh rằng $\sqrt{a}+\sqrt{b}+\sqrt{c}\geq ab+bc+ca$

Bài này hình như là Russia MO 2002 gì đó.

Bất đẳng thức tương đương:

$2\left (\sqrt{a}+\sqrt{b}+\sqrt{c} \right )+a^2+b^2+c^2\geq (a+b+c)^2=9$

BĐT trên đúng vì theo AM - GM ta có: 

$a^2+\sqrt{a}+\sqrt{a}\geq 3a$. Thiết lập các BĐT còn lại ta có điều phải chứng minh.




#706189 [TOPIC] ÔN THI BẤT ĐẲNG THỨC $\boxed{\text{THPT CHUYÊN}}$...

Đã gửi bởi Khoa Linh on 17-04-2018 - 18:39 trong Tài liệu - Đề thi

Bài 6: Cho a, b, c>0 thỏa mãn $abc=1$ Chứng minh rằng:

$a+b+c\geq \frac{1+a}{1+b}+\frac{1+b}{1+c}+\frac{1+c}{1+a}$ (Phạm Kim Hùng)

Bài 7: Cho a, b là hai số không âm thỏa mãn $a^3+b^3=2$. Chứng minh:

$3(a^4+b^4)+2a^4b^4\leq 8$ (Vasile Cirtoaje)

Mình xin đưa ra lời giải hai bài này 

Bài 6:

Ta có: $\frac{a+1}{b+1}=a+1-\frac{b(a+1)}{b+1}$ nên BĐT quy về chứng minh:

$\sum \frac{b(a+1)}{b+1}\geq 3$.

Bất đẳng thức trên đúng theo AM - GM 3 số kết hợp với điều kiện $abc=1$

Bài 7: 

Theo AM - GM ta có:

$a^3+b^3+1\geq 3ab\Rightarrow ab\leq 1$; $a^3+1+1\geq 3a\Rightarrow 3a\leq a^3+2=4-b^3$; $3b\leq 4-a^3$.

Từ đó ta có:

$3(a^4+b^4)+2a^4b^4=3a.a^3+3b.b^3+2a^3b^3.ab\leq (4-b^3)a^3+(4-a^3)b^3+2a^3b^3=4(a^3+b^3)=8$




#706402 [TOPIC] ÔN THI BẤT ĐẲNG THỨC $\boxed{\text{THPT CHUYÊN}}$...

Đã gửi bởi Khoa Linh on 19-04-2018 - 15:21 trong Tài liệu - Đề thi

Bài 40: Cho các số $a,b,c>0$ thoả mãn $a+b+c=3$. Chứng minh rằng $\sum\frac{1}{a^2+2b^2+3}\leq\frac{1}{2}$

Bất đẳng thức này lần trước mình đăng nhưng bị sai nên giả thiết phải là $abc=1$

Khi đó ta có:

$\sum \frac{1}{a^2+2b^2+3}=\sum \frac{1}{(a^2+b^2)+(b^2+1)+2}\leq \frac{1}{2}\sum \frac{1}{ab+b+1}=\frac{1}{2}$




#706528 [TOPIC] ÔN THI BẤT ĐẲNG THỨC $\boxed{\text{THPT CHUYÊN}}$...

Đã gửi bởi Khoa Linh on 20-04-2018 - 19:55 trong Tài liệu - Đề thi

Bài 58

Cho các số thực không âm a, b, c thỏa mãn $a+b+c=3$. Chứng minh:

$\frac{a}{{\sqrt {2a + b} }} + \frac{b}{{\sqrt {2b + c} }} + \frac{c}{{\sqrt {2c + a} }} \le \sqrt 3$

Ta có BĐT tương đương: $\sum \frac{a}{\sqrt{3(2a+b)}}\leq 1$

Mà ta có: $\frac{a}{\sqrt {3(2a + b)}}=\sqrt{\frac{a^2}{3(2a+b)}}\leq \frac{1}{2}\left (\frac{a}{2a+b}+\frac{a}{3} \right )$

Nên BĐT quy về chứng minh: $\sum \frac{a}{2a+b}\leq 1\Leftrightarrow \sum \frac{b}{2a+b}\geq 1$ 

Bất đẳng thức trên đúng theo Cauchy - Schwarz: $\sum \frac{b}{2a+b}=\sum \frac{b^2}{2ab+b^2}\geq \frac{(a+b+c)^2}{(a+b+c)^2}=1$




#706745 [TOPIC] ÔN THI BẤT ĐẲNG THỨC $\boxed{\text{THPT CHUYÊN}}$...

Đã gửi bởi Khoa Linh on 23-04-2018 - 16:57 trong Tài liệu - Đề thi

Bài 77(APMO 2004): Cho a, b, c là các số thực dương. CMR: $(a^2+2)(b^2+2)(c^2+2)\geq 9(ab+bc+ca)$

Cách 2

Áp dụng Dirichlet ta giả sử $(b^2-1)(c^2-1)\geq 0\Leftrightarrow b^2c^2\geq b^2+c^2-1$

Suy ra $(a^2+2)(b^2+2)(c^2+2)=(a^2+2)(b^2c^2+2b^2+2c^2+4)\geq (a^2+2)(3b^2+3c^2+3)=3(a^2+1+1)(1+b^2+c^2)\geq 3(a+b+c)^2$




#706612 [TOPIC] ÔN THI BẤT ĐẲNG THỨC $\boxed{\text{THPT CHUYÊN}}$...

Đã gửi bởi Khoa Linh on 21-04-2018 - 21:37 trong Tài liệu - Đề thi

Bài 66: Cho các số dương a, b, c thỏa mãn $abc\leq 1$. Chứng minh rằng:

$\frac{a}{b}+\frac{b}{c}+\frac{c}{a}\geq a+b+c$

p/s: Bài không khó nhưng đẹp  :D




#706566 [TOPIC] ÔN THI BẤT ĐẲNG THỨC $\boxed{\text{THPT CHUYÊN}}$...

Đã gửi bởi Khoa Linh on 21-04-2018 - 00:27 trong Tài liệu - Đề thi

Bài 61: Cho các số thực x, y, z thỏa mãn $x^2+y^2+z^2=2$.

Chứng minh rằng: $x+y+z\leq xyz+2$

(IMO  Shortlist 1987)




#706565 [TOPIC] ÔN THI BẤT ĐẲNG THỨC $\boxed{\text{THPT CHUYÊN}}$...

Đã gửi bởi Khoa Linh on 21-04-2018 - 00:23 trong Tài liệu - Đề thi

Bài 49(Phạm Kim Hùng): Cho a, b, c dương và $a^2+b^2+c^2=3$. Chứng minh rằng:

$\frac{a}{a^2+2b+3}+\frac{b}{b^2+2c+3}+\frac{c}{c^2+2a+3}\leq \frac{1}{2}$

Bài 50(Phạm Kim Hùng): Cho 4 số dương a, b, c, d có tích bằng 1. Chứng minh:

$(a^2+1)(b^2+1)(c^2+1)(d^2+1)\geq (a+b+c+d)^2$

Mình xin đưa ra lời giải hai bài này.

Bài 49:

$\sum \frac{a}{a^2+2b+3}\leq \frac{1}{2}\sum \frac{a}{a+b+1}$

Bất đẳng thức tương tương: $\sum \frac{a}{a+b+1}\leq 1\Leftrightarrow \sum \frac{b+1}{a+b+1}\geq 2$

Áp dụng Cauchy - Schwarz ta có:

$\sum \frac{b+1}{a+b+1}\geq \frac{(a+b+c+3)^2}{(b+1)(a+b+1)+(c+1)(b+c+1)+(a+1)(c+a+1)}=1$ (khai triển tất cả kết hợp giả thiết $a^2+b^2+c^2=3$)

Bài 50:

Theo nguyên lý Dirichlet thì trong 4 số $a-1;b-1;c-1;d-1$ thì có hai số mà tích của chúng không âm. Giả sử $(b-1)(c-1)\geq 0$

Áp dụng Cauchy - Schwarz ta có:

$(a^2+1)(b^2+1)(c^2+1)(d^2+1)=(a^2c^2+a^2+c^2+1)(b^2d^2+1+b^2+d^2)\geq (abcd+a+bc+d)^2=(1+a+d+bc)^2$

Bất đẳng thức quy về chứng minh: $1+a+d+bc\geq a+b+c+d\Leftrightarrow (b-1)(c-1)\geq 0$ (đúng)




#706753 [TOPIC] ÔN THI BẤT ĐẲNG THỨC $\boxed{\text{THPT CHUYÊN}}$...

Đã gửi bởi Khoa Linh on 23-04-2018 - 17:40 trong Tài liệu - Đề thi

Bài 82: Cho 3 số dương a, b, c thỏa mãn $a+b+c=3$. Chứng minh rằng: 

$\frac{a}{ab+1}+\frac{b}{bc+1}+\frac{c}{ca+1}\geq \frac{3}{2}$

(Sưu tầm)